ABCs to the Max

Algebra Level 3

Find the largest integer N N such that a 2 b 2 c 2 + a b + b c + c a N a b c a^2b^2c^2 + ab + bc + ca \geq N abc for all positive numbers a a , b b , and c c .


The answer is 4.

This section requires Javascript.
You are seeing this because something didn't load right. We suggest you, (a) try refreshing the page, (b) enabling javascript if it is disabled on your browser and, finally, (c) loading the non-javascript version of this page . We're sorry about the hassle.

19 solutions

Michael Tang
Oct 13, 2013

In the inequality, we let a = b = c = 1 a=b=c=1 to bound the possible values of N . N. This gives 4 N . 4 \ge N.

We now prove that N = 4 N = 4 works; by AM-GM, we have

a 2 b 2 c 2 + a b + b c + c a 4 ( a 2 b 2 c 2 ) ( a b ) ( b c ) ( c a ) 4 = a b c , \dfrac{a^2b^2c^2+ab+bc+ca}{4} \ge \sqrt[4]{(a^2b^2c^2)(ab)(bc)(ca)} = abc,

so a 2 b 2 c 2 + a b + b c + c a 4 a b c a^2b^2c^2+ab+bc+ca \ge 4abc for all positive a , b , c . a,b,c. Since 4 N 4 \ge N and N = 4 N = 4 is possible, the maximum is N = 4 . N = \boxed{4}.

Great!

As always, remember to justify why the number you found is indeed the maximum, and not just a potential candidate for the maximum which you were lucky enough to guess.

Calvin Lin Staff - 7 years, 7 months ago
Aaron Doman
Oct 13, 2013

By the AM-GM inequality, we have a 2 b 2 c 2 + a b + b c + c a 4 a 2 b 2 c 2 a b b c c a 4 \frac{a^2b^2c^2+ab+bc+ca}{4} \ge \sqrt[4]{a^2b^2c^2\cdot ab\cdot bc\cdot ca} = a b c . =abc. Thus, N = 4 \boxed{N=4} .

How do you know N can't be higher?

Michael Tang - 7 years, 8 months ago

The question should be what is the least integer of N

Kelvin Ng - 7 years, 8 months ago

N N cannot be larger, since equality holds at some point in AM-GM (namely when a = b = c = 1 a=b=c=1 ).

Aaron Doman - 7 years, 8 months ago

Since i haven't learnt the AM-GM inequality yet,so i dun use it. however,a,b ,c can be any positive numbers .This results in the N will be also different. Let a=1,b=1,c=3,N will be 5.333333.

Kelvin Ng - 7 years, 8 months ago

Log in to reply

It seems to me that your confusion stems from not understanding the question. Try answering this:

Find the largest integer N N such that 4 N 4 \geq N .

and compare it to

Find the smallest integer N N such that 4 N 4 \geq N .

What would be your answers?

Calvin Lin Staff - 7 years, 7 months ago

The N has to hold for all a , b and c . If a = 1, b = 1 and c =3 then

1 2 × 1 2 × 3 2 + 1 × 1 + 1 × 3 + 3 × 1 = 16 1^{2} \times 1^{2} \times 3^{2} + 1 \times 1 + 1 \times 3 + 3 \times 1 = 16

and 16 \beq 4 × 1 × 1 × 3 = 12 16 \beq 4 \times 1 \times 1 \times 3 = 12 ...

Meike Rouwenhorst - 7 years, 8 months ago

Log in to reply

however N can be 5 as well~^^ The question should ask us to find the smallest value of N.

Kelvin Ng - 7 years, 8 months ago

Log in to reply

@Kelvin Ng The problem is correct; it should be the largest N N , because if it were any larger than 4, the inequality would not hold for ( 1 , 1 , 1 ) (1,1,1) .

Aaron Doman - 7 years, 8 months ago

Log in to reply

@Aaron Doman However,the question doesn't state that "a" should be equal to "b" and equal to "c".

Kelvin Ng - 7 years, 8 months ago

Log in to reply

@Kelvin Ng But the way it is phrased does not prohibit a , b , c a,b,c being equal.

Aaron Doman - 7 years, 8 months ago

Log in to reply

@Aaron Doman there are many possible groups of a,b and c....The question should ask the least value of N rather than the largest one because,there are many possible groups of (a,b,c).

Kelvin Ng - 7 years, 8 months ago

Log in to reply

@Kelvin Ng Let's try like this, give me the greatest number N that satisfies the inequality, so when i try every possible combination (a,b,c) it will hold. If you give me N>4, then I can find (a=b=c=1), so it means you were wrong. If you say the least, then N=1; N=2 and N=3, so there're infinite real answers so the problem won't make sense

Daniel Griza - 7 years, 7 months ago

We are interested in finding the minimum possible value of N, which turns out too be 4. so N may be greater than 4 as you found out. Try finding a,b,c such that N is less than 4.(You can't!!!)

Kumar Ashutosh - 7 years, 7 months ago

I think that N can be any numbers as high as possible it can be.

Kelvin Ng - 7 years, 8 months ago

Log in to reply

try a=b=c=1 and N=5

Snehdeep Arora - 7 years, 8 months ago

The problem is correct. The sticking point here is, as you basically figured out, the words "for all". It is not the same as "for some". We are looking for N N that satisfy the given conditions even if the choice of a a , b b and c c is left to our worst enemy (or a monkey with an unlimited access to a typewriter).

Alexander Borisov - 7 years, 7 months ago

As a,b and c values increase,N can be also increased.

Kelvin Ng - 7 years, 8 months ago

Using the AM-GM inequality, we have a b + b c + c a 3 a 2 b 2 c 3 3 {ab+bc+ca \over 3} \ge \sqrt[3]{a^2b^2c^3} . So, we have:

a 2 b 2 c 2 + a b + b c + c a a^2b^2c^2+ab+bc+ca

a 2 b 2 c 2 + 3 a 2 b 2 c 2 3 . . . . ( 1 ) \ge a^2b^2c^2+3\sqrt[3]{a^2b^2c^2} .... (1) .

So if N N satisfies 1 1 , then it also satisfies the original inequality. If N N satisfies 1 1 , then,

a 2 b 2 c 2 + 3 a 2 b 2 c 2 3 N a b c a^2b^2c^2 + 3\sqrt[3]{a^2b^2c^2} \ge Nabc .

p 4 N p + 3 0 \equiv p^4-Np+3 \ge 0 where p = ( a b c ) 1 3 p = (abc)^{1 \over 3} . Let f ( x ) = x 4 N x + 3 f(x) = x^4-Nx+3 . This function has a single real minima at x = ( N 4 ) 1 3 x^\star=\left(N \over 4\right)^{1 \over 3} . If the value of N N is such that f ( x ) 0 f(x^\star) \ge 0 , then ( 1 ) (1) is true for all N N . Substituting x x^\star in f ( x ) = 0 f(x)=0 , and solving for N N , we get N = 4 N=4 . It is easy to verify that the inequality a 2 b 2 c 2 + a b + b c + c a N a b c a^2b^2c^2+ab+bc+ca \ge Nabc is not satisfied for N = 5 N=5 by letting a = b = c = 1 a=b=c=1 and noting that 1 + 3 5 1+3 \ngeq 5 . So N = 4 N=4 is the largest integer that satisfies the inequality.

Kumar Ashutosh
Oct 15, 2013

From AM-GM inequality

We know that AM \geq GM Taking a 2 b 2 c 2 , a b , b c , c a a^2b^2c^2, ab, bc, ca as four terms we get

a 2 b 2 c 2 + a b + b c + c a 4 a 2 b 2 c 2 . a b . b c . c a 4 \frac{a^2b^2c^2+ab+bc+ca}{4} \geq \sqrt[4]{a^2b^2c^2.ab.bc.ca}

a 2 b 2 c 2 + a b + b c + c a 4 a b c \Rightarrow \frac{a^2b^2c^2+ab+bc+ca}{4} \geq abc

a 2 b 2 c 2 + a b + b c + c a 4 a b c \Rightarrow a^2b^2c^2+ab+bc+ca \geq 4abc

Thus N = 4 N= \boxed{4}

Sushil Varma
Oct 14, 2013

Use AM> =GM

Kishlaya Jaiswal
Oct 15, 2013

Using directly AM-GM inequality

a 2 b 2 c 2 + a b + b c + c a 4 ( a 4 b 4 c 4 ) 1 4 = a b c \frac{a^2b^2c^2 + ab + bc + ca}{4} \geq (a^4b^4c^4)^\frac{1}{4} = abc

Therefore, a 2 b 2 c 2 + a b + b c + c a 4 a b c a^2b^2c^2 + ab + bc + ca \geq 4abc

Hence N = 4 N = 4

Snehdeep Arora
Oct 14, 2013

Using A . M G . M A.M \ge G.M inequality:

( a 2 b 2 c 2 + a b + b c + c a ) / 4 ( a 4 b 4 c 4 ) 1 / 4 (a^{2}b^{2}c^{2}+ab+bc+ca)/4 \ge (a^4b^4c^4)^{1/4} or ( a 2 b 2 c 2 + a b + b c + c a ) 4 a b c (a^{2}b^{2}c^{2}+ab+bc+ca) \ge 4abc

Comparing N = 4

Again, how do you know N can't be larger?

Michael Tang - 7 years, 8 months ago

Log in to reply

I tried N=5 putting a,b,c as 1 which clearly do not satisfy.

Snehdeep Arora - 7 years, 8 months ago

Consider a=1, b=1, c=1.
1+1+1+1≥N.
N=4

How do you know there are no other values of a , b , c a,b,c that result in a smaller value of N N ?

Aaron Doman - 7 years, 8 months ago

Log in to reply

That's why the question should ask us to find the smallest value of N instead of the largest one.

Kelvin Ng - 7 years, 8 months ago

Log in to reply

No, the question is fine as it is.

Michael Tang - 7 years, 8 months ago

Log in to reply

@Michael Tang how can u explain this?

Kelvin Ng - 7 years, 8 months ago

If the question was the smallest value of N then what would be you answer ???

Santanu Banerjee - 7 years, 8 months ago
Christopher Boo
Apr 7, 2014

By AM-GM Inequality,

a 2 b 2 c 2 + a b + b c + c a a^2b^2c^2+ab+bc+ca

4 \strut ( a 2 b 2 c 2 ) ( a b ) ( b c ) ( c a ) 4 \geq 4\sqrt[4]{\strut (a^2b^2c^2)(ab)(bc)(ca)}

4 a b c \geq4abc

Hence, the maximum value of N N is 4.

Apoorv Pandey
Mar 23, 2014

This can be done easily by using the AM-GM inequality

Mayankk Bhagat
Jan 27, 2014

it is clear enough that if one puts any value less than 1 like a=0.2,b=0.5,c=0.9 etc......... n is bound to exceed 10. similar is the case with nos. greater than 1 where the value of n goes on increasing with succeeding inputs........ but when we put a=1,b=1,c=1 we deduce n=4........which happens to be the largest integer at which this expression holds good for all positive nos

AM-GM!

Xuming Liang - 7 years, 3 months ago

USE AM>=GM...

Shikhar Jaiswal - 7 years, 2 months ago
Ralph Schraven
Oct 20, 2013

We see that a, b and c are squared on the LHS which furthermore only contains terms increasing the value of the LHS. We see that a, b and c are not squared but linearly multiplied with each other on the RHS. This and the fact that a, b and c are positive integers means that we just need to minimize the values of a, b and c. This means we simply need to plug in ( a , b , c ) = ( 1 , 1 , 1 ) (a,b,c) = (1,1,1) to arrive at the largest integer N. N = ( 1 2 ) ( 1 2 ) ( 1 2 ) + ( 1 ) ( 1 ) + ( 1 ) ( 1 ) + ( 1 ) ( 1 ) = 4 N = (1^{2})(1^{2})(1^{2}) + (1)(1) + (1)(1) + (1)(1) = \boxed{4}

Daniel Ferreira
Oct 17, 2013

O menor valor que as incógnitas podem assumir é 1, daí,

1 2 1 2 1 2 + 1 1 + 1 1 + 1 1 N 1 1 1 1 + 1 + 1 + 1 N 4 N N 4 1^2 \cdot 1^2 \cdot 1^2 + 1 \cdot 1 + 1 \cdot 1 + 1 \cdot 1 \geq N \cdot 1 \cdot 1 \cdot 1 \\\\ 1 + 1 + 1 + 1 \geq N \\\\ 4 \geq N \\\\ N \leq 4

Logo, N = 4 \boxed{N = 4}

Jon K
Oct 16, 2013

By AMGM, note

a b + b c > = 2 b a c ab + bc >= 2b\sqrt{ac}

a b + a c > = 2 a b c ab + ac >= 2a\sqrt{bc}

b c + a c > = 2 c a b bc + ac >= 2c\sqrt{ab}

adding and dividing by 2 gives a b + b c + a c > = b a c + a b c + c a b ab + bc + ac >= b\sqrt{ac} + a\sqrt{bc} + c\sqrt{ab} substituting into the original equation and dividing by a b c abc a b c + 1 / a c + 1 / b c + 1 / a b > = N abc + 1/\sqrt{ac} + 1/\sqrt{bc} + 1/\sqrt{ab} >= N which clearly has a minimum when a = b = c = 1 hence N = 4

Jan J.
Oct 15, 2013

By AM-GM $$a^2b^2c^2 + ab + bc + ca \geq 4\sqrt[4]{a^4b^4c^4} = 4abc$$ On the other hand for ( a , b , c ) = ( 1 , 1 , 1 , 1 ) (a,b,c) = (1,1,1,1) we get $$4 \geq N$$ Hence $$N = \boxed{4}$$

Tushar Gopalka
Oct 14, 2013

a^2 b^2 c^2 +ab+bc+ca >= Nabc.......Dividing each side of inequality by abc....................... As all three numbers a,b,c are positive, sign of inequality remains unchanged. Hence, abc+1/a+1/b+1/c>=N....Hence the greatest value of N is the least value of abc+1/a+1/b+1/c. Applyng AM- GM inequality, we get that abc+1/a+1/b+1/c>=4.......Hence , N max =4

Sam Leo
Oct 14, 2013

a 2 b 2 c 2 + a b + b c + c a a b c = a b c + a + b + c \frac{a^{2}*b^{2}*c^{2}+ab+bc+ca}{abc} = abc+a+b+c

a b c + a + b + c > = N abc+a+b+c>=N

Minimum condition for equation to be true: a,b,c = 1. 1 1 1 + 1 + 1 + 1 = 4 1*1*1+1+1+1 = 4 Therefore, largest integer * N N = 4 *

The first step in your algebra is incorrect -- so even though you got the right solution, you should try this problem again.

Simon Tyler - 7 years, 7 months ago
Santanu Banerjee
Oct 14, 2013

On dividing with abc both sides we get::

abc + (1/a) + (1/b) + (1/c) >= N

Symmetrically a = b = c = 1 should be the solution

ANS :-: 4

what do you mean by "Symmetrically a = b = c = 1 should be the solution" as the question clearly states that a.b and c can take any value

Snehdeep Arora - 7 years, 8 months ago

Log in to reply

Based on maxima minima usually in such cases where there is symmetry there exists either a maxima or minima of the function

Santanu Banerjee - 7 years, 7 months ago

The symmetry argument is often good, but needs to be justified. For example, the triangle of a fixed perimeter with largest area is the symmetric solution, but the triangle of fixed perimeter with the smallest area is the non-symmetric degenerate case.

Simon Tyler - 7 years, 7 months ago
Adrabi Abderrahim
Oct 14, 2013

the largest number N N for all positive numbers a , b , c a, b, c , this mean a = b = c = 1 a = b = c = 1 is included and it's the smallest possible solution for all ( a , b , c ) (a, b, c) then:

a 2 b 2 c 2 + a b + b c + c a = ( 1 2 ) × ( 1 2 ) × ( 1 2 ) + ( 1 ) × ( 1 ) + ( 1 ) × ( 1 ) + ( 1 ) × ( 1 ) = 4 a^2b^2c^2 + ab + bc + ca = (1^2)\times(1^2)\times(1^2) + (1)\times(1) + (1)\times(1) + (1)\times(1) = 4

then 4 N 4 \geq N so N = 4 N = 4 is solution.

0 pending reports

×

Problem Loading...

Note Loading...

Set Loading...